[Date Prev][Date Next][Thread Prev][Thread Next][Date Index][Thread Index]

Re: [obm-l] treino para olimpiadas...






>From: DEOLIVEIRASOU@aol.com
>Reply-To: obm-l@mat.puc-rio.br
>To: obm-l@mat.puc-rio.br
>Subject: [obm-l] treino para olimpiadas...
>Date: Wed, 3 Apr 2002 18:29:03 EST
>
>Quem pode dar uma força nessas pelo menos??
>1)para que valores de n, 5^n+n^6 é divisivel por 13?

Inicialmente note que:
5^2 == - 1 (mod. 13)   =>   5^2k == (-1)^k (mod. 13)   =>
5^4k == 1 (mod. 13)    5^(4k + 1) == 5 (mod. 13)
5^(4k + 2) == - 1 (mod. 13)    5^(4k + 3) == - 5 (mod. 13)

Por 13:
se  n == 0 (mod. 13)   =>   n^6 == 0 (mod. 13)
se  n == +/- 1 (mod. 13)   =>   n^6 == 1 (mod. 13)
se  n == +/- 2 (mod. 13)   =>   n^6 == - 1 (mod. 13)
se  n == +/- 3 (mod. 13)   =>   n^6 == 1 (mod. 13)
se  n == +/- 4 (mod. 13)   =>   n^6 == 1 (mod. 13)
se  n == +/- 5 (mod. 13)   =>   n^6 == - 1 (mod. 13)
se  n == +/- 6 (mod. 13)   =>   n^6 == - 1 (mod. 13)

Pelos valores encontrados, teremos resto 0 quando tivermos um resto 1 de 5^n 
com um - 1 de n^6 ou um resto - 1 de 5^n com um 1 de n^6.
Vejamos as possibilidades:
i) n = 4a  e  n = 13b +/- 2   =>   n = 52k + 24  ou  n = 52k + 28
ii) n = 4a  e  n = 13b +/- 5   =>   n = 52k + 8  ou  n = 52k + 44
iii) n = 4a  e  n = 13b +/- 6   =>   n = 52k + 20  ou  n = 52k + 32
iv) n = 4a + 2  e  n = 13b +/- 1   =>   n = 52k + 38  ou  n = 52k + 14
v) n = 4a + 2  e  n = 13b +/- 3   =>   n = 52k + 10  ou  n = 52k + 42
vi) n = 4a + 2  e  n = 13b +/- 4   =>   n = 52k + 30  ou  n = 52k + 22

Salvo algum erro de conta acredito que esteja correto.



>2) Existem inteiros m e n tais que 5m^2-6mn+7n^2=1985??
>3)(IMO-1976)Determine, com prova, o maior número queé o produto de inteiros 
>positivos cuja soma é 1976.

Como  1976  é par, poderíamos imaginar que a decomposição de 1976 como soma 
de inteiros positivos que possui o maior produto seja  1976 = 2 + 2 + 2 + 2 
+ ... + 2,  onde temos 988 2's. Entretanto, notemos que se no lugar da soma 
de três números dois (2 + 2 + 2 = 6) escrevermos  3 + 3 (= 6), temos que  
2.2.2 < 3.3 (8 < 9), onde concluímos que devemos substituir cada conjunto de 
3 números 2 por 2 números 3 para maximizar o produto. Se fizermos o mesmo 
para 4, notamos que desta vez não seria melhor substituir 4 números 3 por 3 
números 4, pois  3.3.3.3 > 4.4.4 (81 > 64), o mesmo raciocínio valendo para 
5, uma vez que  3.3.3.3.3 > 5.5.5 (243 > 125). Desta forma, concluímos que a 
decomposição de qualquer inteiro n como soma de inteiros positivos tal que o 
produto destes inteiros seja o maior possível deve possuir o maior número 
possível de 3's, completando com 2's (se necessário).
Assim, podemos separar em 3 casos:
  i) se n = 3k:  n = 3 + 3 + 3 + ... + 3   =>   Pn = 3^k
ii) se n = 3k + 1:  n = 3 + 3 + 3 + ... + 3 + 2 + 2   =>
Pn = 4.3^(k – 1)
iii) se n = 3k + 2:  n = 3 + 3 + 3 + ... + 3 + 2   =>   Pn = 2.3^k
Como  1976 = 3.658 + 2, a decomposição de 1976 como soma de inteiros 
positivos que possui maior produto deste inteiros é  1976 = 3 + 3 + 3 + 3 + 
... + 3 + 3 + 2,  onde temos 658 números 3, e o produto é igual a P1976 = 
2.3^658.


Até mais,
Marcelo Rufino de Oliveira

_________________________________________________________________
Join the world’s largest e-mail service with MSN Hotmail. 
http://www.hotmail.com

=========================================================================
Instruções para entrar na lista, sair da lista e usar a lista em
http://www.mat.puc-rio.br/~nicolau/olimp/obm-l.html
O administrador desta lista é <nicolau@mat.puc-rio.br>
=========================================================================